論文よりa=Π[k=1,r](1+pk+…pk^qk)
この前提を踏まえてなお
>a'=Π[k=1,r-1](1+pk+…pk^qk)
>となり、a'=a/pk^qk
と言ってるのなら1は相当なお馬鹿さんだ

ここで誤りを認めるなら「お馬鹿さん」は撤回してもいいが。